Prove the alternating sum of a decreasing sequence converging to 0 is Cauchy. Unicorn Meta Zoo...

Israeli soda type drink

Has a Nobel Peace laureate ever been accused of war crimes?

Is Electric Central Heating worth it if using Solar Panels?

What is this word supposed to be?

Second order approximation of the loss function (Deep learning book, 7.33)

Would reducing the reference voltage of an ADC have any effect on accuracy?

Check if a string is entirely made of the same substring

Could Neutrino technically as side-effect, incentivize centralization of the bitcoin network?

Prove the alternating sum of a decreasing sequence converging to 0 is Cauchy.

How to open locks without disable device?

Did the Roman Empire have penal colonies?

My bank got bought out, am I now going to have to start filing tax returns in a different state?

Why does the Cisco show run command not show the full version, while the show version command does?

PIC mathematical operations weird problem

With indentation set to `0em`, when using a line break, there is still an indentation of a size of a space

As an international instructor, should I openly talk about my accent?

Raising a bilingual kid. When should we introduce the majority language?

A strange hotel

What is a 'Key' in computer science?

Co-worker works way more than he should

Rolling Stones Sway guitar solo chord function

Is Bran literally the world's memory?

What is /etc/mtab in Linux?

Can I criticise the more senior developers around me for not writing clean code?



Prove the alternating sum of a decreasing sequence converging to 0 is Cauchy.



Unicorn Meta Zoo #1: Why another podcast?
Announcing the arrival of Valued Associate #679: Cesar ManaraSuppose for all positive integers $n$, $|x_n-y_n|< frac{1}{n}$ Prove that $(x_n)$ is also Cauchy.Proof check for completenessProve that $d_n$ is a Cauchy sequence in $mathbb{R}$Prove ${aX_n +bY_n}$ is a Cauchy Sequence.Prove a sequence is a Cauchy and thus convergentIf $(x_n)$ and $(y_n)$ are Cauchy sequences, then give a direct argument that $ (x_n + y_n)$ is a Cauchy sequenceIf ${x_n}$ and ${y_n}$ are Cauchy then $left{frac{2x_n}{y_n}right}$ is CauchyLet ${x_n}$ be a Cauchy sequence of rational numbers. Define a new sequence ${y_n}$ by $y_n = (x_n)(x_{n+1})$. Show that ${y_n}$ is a CS.Let ${x_n}$ be a Cauchy sequence of real numbers, prove that a new sequence ${y_n}$, with $y_n$=$x_n^frac{1}{3}$, is also a Cauchy sequence.$x_n rightarrow x$ iff the modified sequence is Cauchy












2












$begingroup$


Let $(x_n)$ be a decreasing sequence with $x_n > 0$ for all $n in mathbb{N}$, and $(x_n) to 0$. Let $(y_n)$ be defined for all $n in mathbb{N}$ by
$$y_n = x_0 - x_1 + x_2 - cdots + (-1)^n x_n .$$



I want to show, using the $varepsilon$ definition, that $(y_n)$ is Cauchy.



I am trying to find, given $varepsilon > 0$, a real number $N$ such that for all $m$ and $n$ with $m > n > N$, $|y_m - y_n| < varepsilon$.



I have been going backwards to try and find $N$, and have
begin{align*}
|y_m - y_n| & = left| (x_0 - x_1 + cdots pm x_m) - (x_0 - x_1 + cdots pm x_n) right| \
|y_m - y_n| & = left| x_{n + 1} - x_{n + 2} + cdots pm x_{m} right| \
|y_m - y_n| & leq | x_{n + 1} | + | x_{n + 2} | + cdots + | x_{m} | \
|y_m - y_n| & leq ?
end{align*}



I do not know how to get a solution from there, and am not sure about the process, particurlary the last step since I feel getting rid of the minuses might prevent me from finding a solution.










share|cite|improve this question









$endgroup$








  • 1




    $begingroup$
    Because the series is alternating and decreasing, I think you can prove by induction on $m$ that $|y_m-y_n| leq |y_n|$.
    $endgroup$
    – Robert Shore
    3 hours ago










  • $begingroup$
    @RobertShore is my answer okay?
    $endgroup$
    – Subhasis Biswas
    3 hours ago










  • $begingroup$
    @RobertShore yes I can definitely show that, but it brings me to the same issue with $|y_m| leq |x_0 - x_1 + cdots pm x_m|$, and I am unsure how to proceed from there.
    $endgroup$
    – oranji
    42 mins ago


















2












$begingroup$


Let $(x_n)$ be a decreasing sequence with $x_n > 0$ for all $n in mathbb{N}$, and $(x_n) to 0$. Let $(y_n)$ be defined for all $n in mathbb{N}$ by
$$y_n = x_0 - x_1 + x_2 - cdots + (-1)^n x_n .$$



I want to show, using the $varepsilon$ definition, that $(y_n)$ is Cauchy.



I am trying to find, given $varepsilon > 0$, a real number $N$ such that for all $m$ and $n$ with $m > n > N$, $|y_m - y_n| < varepsilon$.



I have been going backwards to try and find $N$, and have
begin{align*}
|y_m - y_n| & = left| (x_0 - x_1 + cdots pm x_m) - (x_0 - x_1 + cdots pm x_n) right| \
|y_m - y_n| & = left| x_{n + 1} - x_{n + 2} + cdots pm x_{m} right| \
|y_m - y_n| & leq | x_{n + 1} | + | x_{n + 2} | + cdots + | x_{m} | \
|y_m - y_n| & leq ?
end{align*}



I do not know how to get a solution from there, and am not sure about the process, particurlary the last step since I feel getting rid of the minuses might prevent me from finding a solution.










share|cite|improve this question









$endgroup$








  • 1




    $begingroup$
    Because the series is alternating and decreasing, I think you can prove by induction on $m$ that $|y_m-y_n| leq |y_n|$.
    $endgroup$
    – Robert Shore
    3 hours ago










  • $begingroup$
    @RobertShore is my answer okay?
    $endgroup$
    – Subhasis Biswas
    3 hours ago










  • $begingroup$
    @RobertShore yes I can definitely show that, but it brings me to the same issue with $|y_m| leq |x_0 - x_1 + cdots pm x_m|$, and I am unsure how to proceed from there.
    $endgroup$
    – oranji
    42 mins ago
















2












2








2


1



$begingroup$


Let $(x_n)$ be a decreasing sequence with $x_n > 0$ for all $n in mathbb{N}$, and $(x_n) to 0$. Let $(y_n)$ be defined for all $n in mathbb{N}$ by
$$y_n = x_0 - x_1 + x_2 - cdots + (-1)^n x_n .$$



I want to show, using the $varepsilon$ definition, that $(y_n)$ is Cauchy.



I am trying to find, given $varepsilon > 0$, a real number $N$ such that for all $m$ and $n$ with $m > n > N$, $|y_m - y_n| < varepsilon$.



I have been going backwards to try and find $N$, and have
begin{align*}
|y_m - y_n| & = left| (x_0 - x_1 + cdots pm x_m) - (x_0 - x_1 + cdots pm x_n) right| \
|y_m - y_n| & = left| x_{n + 1} - x_{n + 2} + cdots pm x_{m} right| \
|y_m - y_n| & leq | x_{n + 1} | + | x_{n + 2} | + cdots + | x_{m} | \
|y_m - y_n| & leq ?
end{align*}



I do not know how to get a solution from there, and am not sure about the process, particurlary the last step since I feel getting rid of the minuses might prevent me from finding a solution.










share|cite|improve this question









$endgroup$




Let $(x_n)$ be a decreasing sequence with $x_n > 0$ for all $n in mathbb{N}$, and $(x_n) to 0$. Let $(y_n)$ be defined for all $n in mathbb{N}$ by
$$y_n = x_0 - x_1 + x_2 - cdots + (-1)^n x_n .$$



I want to show, using the $varepsilon$ definition, that $(y_n)$ is Cauchy.



I am trying to find, given $varepsilon > 0$, a real number $N$ such that for all $m$ and $n$ with $m > n > N$, $|y_m - y_n| < varepsilon$.



I have been going backwards to try and find $N$, and have
begin{align*}
|y_m - y_n| & = left| (x_0 - x_1 + cdots pm x_m) - (x_0 - x_1 + cdots pm x_n) right| \
|y_m - y_n| & = left| x_{n + 1} - x_{n + 2} + cdots pm x_{m} right| \
|y_m - y_n| & leq | x_{n + 1} | + | x_{n + 2} | + cdots + | x_{m} | \
|y_m - y_n| & leq ?
end{align*}



I do not know how to get a solution from there, and am not sure about the process, particurlary the last step since I feel getting rid of the minuses might prevent me from finding a solution.







real-analysis cauchy-sequences






share|cite|improve this question













share|cite|improve this question











share|cite|improve this question




share|cite|improve this question










asked 3 hours ago









oranjioranji

616




616








  • 1




    $begingroup$
    Because the series is alternating and decreasing, I think you can prove by induction on $m$ that $|y_m-y_n| leq |y_n|$.
    $endgroup$
    – Robert Shore
    3 hours ago










  • $begingroup$
    @RobertShore is my answer okay?
    $endgroup$
    – Subhasis Biswas
    3 hours ago










  • $begingroup$
    @RobertShore yes I can definitely show that, but it brings me to the same issue with $|y_m| leq |x_0 - x_1 + cdots pm x_m|$, and I am unsure how to proceed from there.
    $endgroup$
    – oranji
    42 mins ago
















  • 1




    $begingroup$
    Because the series is alternating and decreasing, I think you can prove by induction on $m$ that $|y_m-y_n| leq |y_n|$.
    $endgroup$
    – Robert Shore
    3 hours ago










  • $begingroup$
    @RobertShore is my answer okay?
    $endgroup$
    – Subhasis Biswas
    3 hours ago










  • $begingroup$
    @RobertShore yes I can definitely show that, but it brings me to the same issue with $|y_m| leq |x_0 - x_1 + cdots pm x_m|$, and I am unsure how to proceed from there.
    $endgroup$
    – oranji
    42 mins ago










1




1




$begingroup$
Because the series is alternating and decreasing, I think you can prove by induction on $m$ that $|y_m-y_n| leq |y_n|$.
$endgroup$
– Robert Shore
3 hours ago




$begingroup$
Because the series is alternating and decreasing, I think you can prove by induction on $m$ that $|y_m-y_n| leq |y_n|$.
$endgroup$
– Robert Shore
3 hours ago












$begingroup$
@RobertShore is my answer okay?
$endgroup$
– Subhasis Biswas
3 hours ago




$begingroup$
@RobertShore is my answer okay?
$endgroup$
– Subhasis Biswas
3 hours ago












$begingroup$
@RobertShore yes I can definitely show that, but it brings me to the same issue with $|y_m| leq |x_0 - x_1 + cdots pm x_m|$, and I am unsure how to proceed from there.
$endgroup$
– oranji
42 mins ago






$begingroup$
@RobertShore yes I can definitely show that, but it brings me to the same issue with $|y_m| leq |x_0 - x_1 + cdots pm x_m|$, and I am unsure how to proceed from there.
$endgroup$
– oranji
42 mins ago












2 Answers
2






active

oldest

votes


















2












$begingroup$

To see that the sequence of partial sums is Cauchy, you cannot use the triangle inequality directly as you did. A famous counter example here is $sum_{k=1}^{infty}frac{(-1)^k}{k}$.



What you can do is grouping the terms of the partial sums $s_n= sum_{j=1}^n(-1)^jx_j$ as follows:




  • Let $m = n+k, k,n in mathbb{N}$


Now, you can write $|s_{m} - s_n|$ in two different ways:



$$|s_{n+k} - s_n| = begin{cases}
|x_{n+1} - (x_{n+2}-x_{n+3}) - cdots - (x_{n+2i}-x_{n+2i+1})| & k = 2i+1 \
|x_{n+1} - (x_{n+2}-x_{n+3}) - cdots - (x_{n+2i-2}-x_{n+2i-1}) - x_{2i}| & k = 2i \
end{cases}
$$



$$|s_{n+k} - s_n| = begin{cases}
|(x_{n+1} - x_{n+2}) + cdots + (x_{n+2i-1}-x_{n+2i}) + x_{n+2i+1}| & k = 2i+1 \
|(x_{n+1} - x_{n+2}) + cdots + (x_{n+2i-1}-x_{n+2i}) | & k = 2i \
end{cases}
$$



Using the fact that $x_n searrow 0$, it follows immediately that for all $k in mathbb{N}$ holds
$$|s_{n+k} - s_n| leq x_{n+1}$$



Hence, for $epsilon > 0$ choose $N_{epsilon}$ such that $x_{N_{epsilon}} < epsilon$. Then, for all $m> n > N_{epsilon}$ you have
$$|s_{m} - s_n| leq x_{n+1} leq x_{N_{epsilon}} < epsilon$$






share|cite|improve this answer











$endgroup$













  • $begingroup$
    This is exactly what I was about to do.
    $endgroup$
    – Subhasis Biswas
    41 mins ago






  • 1




    $begingroup$
    @SubhasisBiswas So, I did it for you :-D
    $endgroup$
    – trancelocation
    39 mins ago



















2












$begingroup$

This is also known as the "Leibnitz's Test".



We write $s_n = x_1-x_2+x_3-...+(-1)^{n+1}x_n$



$s_{2n+2}-s_{2n}=u_{2n+1}-u_{2n+2} geq0$ for all $n$.



$s_{2n+1}-s_{2n-1}=-u_{2n}+u_{2n+1} leq 0$



$s_{2n} =u_1 -(u_2-u_3)-(u_4-u_5)...-u_{2n} leq u_1$, i.e. a monotone increasing sequence bounded above.



$s_{2n+1} =(u_1 -u_2)+(u_3-u_4)+...+u_{2n+1} geq u_1-u_2$, i.e. a monotone decreasing sequence bounded below.



Hence, both are convergent subsequences of $(s_n)$. But, we have $lim (s_{2n+1}-s_{2n})=u_{2n+1}=0$, therefore, they converge to the same limit.



Hence, $(s_n)$ converges, i.e. it is Cauchy.



Note: We conclude that $(s_n)$ converges because the indices of the two subsequences $(s_{2n})$ and $(s_{2n+1})$ i.e. $U ={ 2n+1 : n in mathbb{N}}$ and $V ={ 2n : n in mathbb{N}}$ form a partition of $mathbb{N}$ and they both converge to the same limit.






share|cite|improve this answer









$endgroup$













  • $begingroup$
    I want to use the $varepsilon$ definition of a Cauchy sequence, and not the fact that all convergent sequences are Cauchy, which is why I cannot use this solution.
    $endgroup$
    – oranji
    44 mins ago










  • $begingroup$
    I'll edit this answer.
    $endgroup$
    – Subhasis Biswas
    42 mins ago












Your Answer








StackExchange.ready(function() {
var channelOptions = {
tags: "".split(" "),
id: "69"
};
initTagRenderer("".split(" "), "".split(" "), channelOptions);

StackExchange.using("externalEditor", function() {
// Have to fire editor after snippets, if snippets enabled
if (StackExchange.settings.snippets.snippetsEnabled) {
StackExchange.using("snippets", function() {
createEditor();
});
}
else {
createEditor();
}
});

function createEditor() {
StackExchange.prepareEditor({
heartbeatType: 'answer',
autoActivateHeartbeat: false,
convertImagesToLinks: true,
noModals: true,
showLowRepImageUploadWarning: true,
reputationToPostImages: 10,
bindNavPrevention: true,
postfix: "",
imageUploader: {
brandingHtml: "Powered by u003ca class="icon-imgur-white" href="https://imgur.com/"u003eu003c/au003e",
contentPolicyHtml: "User contributions licensed under u003ca href="https://creativecommons.org/licenses/by-sa/3.0/"u003ecc by-sa 3.0 with attribution requiredu003c/au003e u003ca href="https://stackoverflow.com/legal/content-policy"u003e(content policy)u003c/au003e",
allowUrls: true
},
noCode: true, onDemand: true,
discardSelector: ".discard-answer"
,immediatelyShowMarkdownHelp:true
});


}
});














draft saved

draft discarded


















StackExchange.ready(
function () {
StackExchange.openid.initPostLogin('.new-post-login', 'https%3a%2f%2fmath.stackexchange.com%2fquestions%2f3201256%2fprove-the-alternating-sum-of-a-decreasing-sequence-converging-to-0-is-cauchy%23new-answer', 'question_page');
}
);

Post as a guest















Required, but never shown

























2 Answers
2






active

oldest

votes








2 Answers
2






active

oldest

votes









active

oldest

votes






active

oldest

votes









2












$begingroup$

To see that the sequence of partial sums is Cauchy, you cannot use the triangle inequality directly as you did. A famous counter example here is $sum_{k=1}^{infty}frac{(-1)^k}{k}$.



What you can do is grouping the terms of the partial sums $s_n= sum_{j=1}^n(-1)^jx_j$ as follows:




  • Let $m = n+k, k,n in mathbb{N}$


Now, you can write $|s_{m} - s_n|$ in two different ways:



$$|s_{n+k} - s_n| = begin{cases}
|x_{n+1} - (x_{n+2}-x_{n+3}) - cdots - (x_{n+2i}-x_{n+2i+1})| & k = 2i+1 \
|x_{n+1} - (x_{n+2}-x_{n+3}) - cdots - (x_{n+2i-2}-x_{n+2i-1}) - x_{2i}| & k = 2i \
end{cases}
$$



$$|s_{n+k} - s_n| = begin{cases}
|(x_{n+1} - x_{n+2}) + cdots + (x_{n+2i-1}-x_{n+2i}) + x_{n+2i+1}| & k = 2i+1 \
|(x_{n+1} - x_{n+2}) + cdots + (x_{n+2i-1}-x_{n+2i}) | & k = 2i \
end{cases}
$$



Using the fact that $x_n searrow 0$, it follows immediately that for all $k in mathbb{N}$ holds
$$|s_{n+k} - s_n| leq x_{n+1}$$



Hence, for $epsilon > 0$ choose $N_{epsilon}$ such that $x_{N_{epsilon}} < epsilon$. Then, for all $m> n > N_{epsilon}$ you have
$$|s_{m} - s_n| leq x_{n+1} leq x_{N_{epsilon}} < epsilon$$






share|cite|improve this answer











$endgroup$













  • $begingroup$
    This is exactly what I was about to do.
    $endgroup$
    – Subhasis Biswas
    41 mins ago






  • 1




    $begingroup$
    @SubhasisBiswas So, I did it for you :-D
    $endgroup$
    – trancelocation
    39 mins ago
















2












$begingroup$

To see that the sequence of partial sums is Cauchy, you cannot use the triangle inequality directly as you did. A famous counter example here is $sum_{k=1}^{infty}frac{(-1)^k}{k}$.



What you can do is grouping the terms of the partial sums $s_n= sum_{j=1}^n(-1)^jx_j$ as follows:




  • Let $m = n+k, k,n in mathbb{N}$


Now, you can write $|s_{m} - s_n|$ in two different ways:



$$|s_{n+k} - s_n| = begin{cases}
|x_{n+1} - (x_{n+2}-x_{n+3}) - cdots - (x_{n+2i}-x_{n+2i+1})| & k = 2i+1 \
|x_{n+1} - (x_{n+2}-x_{n+3}) - cdots - (x_{n+2i-2}-x_{n+2i-1}) - x_{2i}| & k = 2i \
end{cases}
$$



$$|s_{n+k} - s_n| = begin{cases}
|(x_{n+1} - x_{n+2}) + cdots + (x_{n+2i-1}-x_{n+2i}) + x_{n+2i+1}| & k = 2i+1 \
|(x_{n+1} - x_{n+2}) + cdots + (x_{n+2i-1}-x_{n+2i}) | & k = 2i \
end{cases}
$$



Using the fact that $x_n searrow 0$, it follows immediately that for all $k in mathbb{N}$ holds
$$|s_{n+k} - s_n| leq x_{n+1}$$



Hence, for $epsilon > 0$ choose $N_{epsilon}$ such that $x_{N_{epsilon}} < epsilon$. Then, for all $m> n > N_{epsilon}$ you have
$$|s_{m} - s_n| leq x_{n+1} leq x_{N_{epsilon}} < epsilon$$






share|cite|improve this answer











$endgroup$













  • $begingroup$
    This is exactly what I was about to do.
    $endgroup$
    – Subhasis Biswas
    41 mins ago






  • 1




    $begingroup$
    @SubhasisBiswas So, I did it for you :-D
    $endgroup$
    – trancelocation
    39 mins ago














2












2








2





$begingroup$

To see that the sequence of partial sums is Cauchy, you cannot use the triangle inequality directly as you did. A famous counter example here is $sum_{k=1}^{infty}frac{(-1)^k}{k}$.



What you can do is grouping the terms of the partial sums $s_n= sum_{j=1}^n(-1)^jx_j$ as follows:




  • Let $m = n+k, k,n in mathbb{N}$


Now, you can write $|s_{m} - s_n|$ in two different ways:



$$|s_{n+k} - s_n| = begin{cases}
|x_{n+1} - (x_{n+2}-x_{n+3}) - cdots - (x_{n+2i}-x_{n+2i+1})| & k = 2i+1 \
|x_{n+1} - (x_{n+2}-x_{n+3}) - cdots - (x_{n+2i-2}-x_{n+2i-1}) - x_{2i}| & k = 2i \
end{cases}
$$



$$|s_{n+k} - s_n| = begin{cases}
|(x_{n+1} - x_{n+2}) + cdots + (x_{n+2i-1}-x_{n+2i}) + x_{n+2i+1}| & k = 2i+1 \
|(x_{n+1} - x_{n+2}) + cdots + (x_{n+2i-1}-x_{n+2i}) | & k = 2i \
end{cases}
$$



Using the fact that $x_n searrow 0$, it follows immediately that for all $k in mathbb{N}$ holds
$$|s_{n+k} - s_n| leq x_{n+1}$$



Hence, for $epsilon > 0$ choose $N_{epsilon}$ such that $x_{N_{epsilon}} < epsilon$. Then, for all $m> n > N_{epsilon}$ you have
$$|s_{m} - s_n| leq x_{n+1} leq x_{N_{epsilon}} < epsilon$$






share|cite|improve this answer











$endgroup$



To see that the sequence of partial sums is Cauchy, you cannot use the triangle inequality directly as you did. A famous counter example here is $sum_{k=1}^{infty}frac{(-1)^k}{k}$.



What you can do is grouping the terms of the partial sums $s_n= sum_{j=1}^n(-1)^jx_j$ as follows:




  • Let $m = n+k, k,n in mathbb{N}$


Now, you can write $|s_{m} - s_n|$ in two different ways:



$$|s_{n+k} - s_n| = begin{cases}
|x_{n+1} - (x_{n+2}-x_{n+3}) - cdots - (x_{n+2i}-x_{n+2i+1})| & k = 2i+1 \
|x_{n+1} - (x_{n+2}-x_{n+3}) - cdots - (x_{n+2i-2}-x_{n+2i-1}) - x_{2i}| & k = 2i \
end{cases}
$$



$$|s_{n+k} - s_n| = begin{cases}
|(x_{n+1} - x_{n+2}) + cdots + (x_{n+2i-1}-x_{n+2i}) + x_{n+2i+1}| & k = 2i+1 \
|(x_{n+1} - x_{n+2}) + cdots + (x_{n+2i-1}-x_{n+2i}) | & k = 2i \
end{cases}
$$



Using the fact that $x_n searrow 0$, it follows immediately that for all $k in mathbb{N}$ holds
$$|s_{n+k} - s_n| leq x_{n+1}$$



Hence, for $epsilon > 0$ choose $N_{epsilon}$ such that $x_{N_{epsilon}} < epsilon$. Then, for all $m> n > N_{epsilon}$ you have
$$|s_{m} - s_n| leq x_{n+1} leq x_{N_{epsilon}} < epsilon$$







share|cite|improve this answer














share|cite|improve this answer



share|cite|improve this answer








edited 24 mins ago

























answered 41 mins ago









trancelocationtrancelocation

14.6k1929




14.6k1929












  • $begingroup$
    This is exactly what I was about to do.
    $endgroup$
    – Subhasis Biswas
    41 mins ago






  • 1




    $begingroup$
    @SubhasisBiswas So, I did it for you :-D
    $endgroup$
    – trancelocation
    39 mins ago


















  • $begingroup$
    This is exactly what I was about to do.
    $endgroup$
    – Subhasis Biswas
    41 mins ago






  • 1




    $begingroup$
    @SubhasisBiswas So, I did it for you :-D
    $endgroup$
    – trancelocation
    39 mins ago
















$begingroup$
This is exactly what I was about to do.
$endgroup$
– Subhasis Biswas
41 mins ago




$begingroup$
This is exactly what I was about to do.
$endgroup$
– Subhasis Biswas
41 mins ago




1




1




$begingroup$
@SubhasisBiswas So, I did it for you :-D
$endgroup$
– trancelocation
39 mins ago




$begingroup$
@SubhasisBiswas So, I did it for you :-D
$endgroup$
– trancelocation
39 mins ago











2












$begingroup$

This is also known as the "Leibnitz's Test".



We write $s_n = x_1-x_2+x_3-...+(-1)^{n+1}x_n$



$s_{2n+2}-s_{2n}=u_{2n+1}-u_{2n+2} geq0$ for all $n$.



$s_{2n+1}-s_{2n-1}=-u_{2n}+u_{2n+1} leq 0$



$s_{2n} =u_1 -(u_2-u_3)-(u_4-u_5)...-u_{2n} leq u_1$, i.e. a monotone increasing sequence bounded above.



$s_{2n+1} =(u_1 -u_2)+(u_3-u_4)+...+u_{2n+1} geq u_1-u_2$, i.e. a monotone decreasing sequence bounded below.



Hence, both are convergent subsequences of $(s_n)$. But, we have $lim (s_{2n+1}-s_{2n})=u_{2n+1}=0$, therefore, they converge to the same limit.



Hence, $(s_n)$ converges, i.e. it is Cauchy.



Note: We conclude that $(s_n)$ converges because the indices of the two subsequences $(s_{2n})$ and $(s_{2n+1})$ i.e. $U ={ 2n+1 : n in mathbb{N}}$ and $V ={ 2n : n in mathbb{N}}$ form a partition of $mathbb{N}$ and they both converge to the same limit.






share|cite|improve this answer









$endgroup$













  • $begingroup$
    I want to use the $varepsilon$ definition of a Cauchy sequence, and not the fact that all convergent sequences are Cauchy, which is why I cannot use this solution.
    $endgroup$
    – oranji
    44 mins ago










  • $begingroup$
    I'll edit this answer.
    $endgroup$
    – Subhasis Biswas
    42 mins ago
















2












$begingroup$

This is also known as the "Leibnitz's Test".



We write $s_n = x_1-x_2+x_3-...+(-1)^{n+1}x_n$



$s_{2n+2}-s_{2n}=u_{2n+1}-u_{2n+2} geq0$ for all $n$.



$s_{2n+1}-s_{2n-1}=-u_{2n}+u_{2n+1} leq 0$



$s_{2n} =u_1 -(u_2-u_3)-(u_4-u_5)...-u_{2n} leq u_1$, i.e. a monotone increasing sequence bounded above.



$s_{2n+1} =(u_1 -u_2)+(u_3-u_4)+...+u_{2n+1} geq u_1-u_2$, i.e. a monotone decreasing sequence bounded below.



Hence, both are convergent subsequences of $(s_n)$. But, we have $lim (s_{2n+1}-s_{2n})=u_{2n+1}=0$, therefore, they converge to the same limit.



Hence, $(s_n)$ converges, i.e. it is Cauchy.



Note: We conclude that $(s_n)$ converges because the indices of the two subsequences $(s_{2n})$ and $(s_{2n+1})$ i.e. $U ={ 2n+1 : n in mathbb{N}}$ and $V ={ 2n : n in mathbb{N}}$ form a partition of $mathbb{N}$ and they both converge to the same limit.






share|cite|improve this answer









$endgroup$













  • $begingroup$
    I want to use the $varepsilon$ definition of a Cauchy sequence, and not the fact that all convergent sequences are Cauchy, which is why I cannot use this solution.
    $endgroup$
    – oranji
    44 mins ago










  • $begingroup$
    I'll edit this answer.
    $endgroup$
    – Subhasis Biswas
    42 mins ago














2












2








2





$begingroup$

This is also known as the "Leibnitz's Test".



We write $s_n = x_1-x_2+x_3-...+(-1)^{n+1}x_n$



$s_{2n+2}-s_{2n}=u_{2n+1}-u_{2n+2} geq0$ for all $n$.



$s_{2n+1}-s_{2n-1}=-u_{2n}+u_{2n+1} leq 0$



$s_{2n} =u_1 -(u_2-u_3)-(u_4-u_5)...-u_{2n} leq u_1$, i.e. a monotone increasing sequence bounded above.



$s_{2n+1} =(u_1 -u_2)+(u_3-u_4)+...+u_{2n+1} geq u_1-u_2$, i.e. a monotone decreasing sequence bounded below.



Hence, both are convergent subsequences of $(s_n)$. But, we have $lim (s_{2n+1}-s_{2n})=u_{2n+1}=0$, therefore, they converge to the same limit.



Hence, $(s_n)$ converges, i.e. it is Cauchy.



Note: We conclude that $(s_n)$ converges because the indices of the two subsequences $(s_{2n})$ and $(s_{2n+1})$ i.e. $U ={ 2n+1 : n in mathbb{N}}$ and $V ={ 2n : n in mathbb{N}}$ form a partition of $mathbb{N}$ and they both converge to the same limit.






share|cite|improve this answer









$endgroup$



This is also known as the "Leibnitz's Test".



We write $s_n = x_1-x_2+x_3-...+(-1)^{n+1}x_n$



$s_{2n+2}-s_{2n}=u_{2n+1}-u_{2n+2} geq0$ for all $n$.



$s_{2n+1}-s_{2n-1}=-u_{2n}+u_{2n+1} leq 0$



$s_{2n} =u_1 -(u_2-u_3)-(u_4-u_5)...-u_{2n} leq u_1$, i.e. a monotone increasing sequence bounded above.



$s_{2n+1} =(u_1 -u_2)+(u_3-u_4)+...+u_{2n+1} geq u_1-u_2$, i.e. a monotone decreasing sequence bounded below.



Hence, both are convergent subsequences of $(s_n)$. But, we have $lim (s_{2n+1}-s_{2n})=u_{2n+1}=0$, therefore, they converge to the same limit.



Hence, $(s_n)$ converges, i.e. it is Cauchy.



Note: We conclude that $(s_n)$ converges because the indices of the two subsequences $(s_{2n})$ and $(s_{2n+1})$ i.e. $U ={ 2n+1 : n in mathbb{N}}$ and $V ={ 2n : n in mathbb{N}}$ form a partition of $mathbb{N}$ and they both converge to the same limit.







share|cite|improve this answer












share|cite|improve this answer



share|cite|improve this answer










answered 3 hours ago









Subhasis BiswasSubhasis Biswas

608512




608512












  • $begingroup$
    I want to use the $varepsilon$ definition of a Cauchy sequence, and not the fact that all convergent sequences are Cauchy, which is why I cannot use this solution.
    $endgroup$
    – oranji
    44 mins ago










  • $begingroup$
    I'll edit this answer.
    $endgroup$
    – Subhasis Biswas
    42 mins ago


















  • $begingroup$
    I want to use the $varepsilon$ definition of a Cauchy sequence, and not the fact that all convergent sequences are Cauchy, which is why I cannot use this solution.
    $endgroup$
    – oranji
    44 mins ago










  • $begingroup$
    I'll edit this answer.
    $endgroup$
    – Subhasis Biswas
    42 mins ago
















$begingroup$
I want to use the $varepsilon$ definition of a Cauchy sequence, and not the fact that all convergent sequences are Cauchy, which is why I cannot use this solution.
$endgroup$
– oranji
44 mins ago




$begingroup$
I want to use the $varepsilon$ definition of a Cauchy sequence, and not the fact that all convergent sequences are Cauchy, which is why I cannot use this solution.
$endgroup$
– oranji
44 mins ago












$begingroup$
I'll edit this answer.
$endgroup$
– Subhasis Biswas
42 mins ago




$begingroup$
I'll edit this answer.
$endgroup$
– Subhasis Biswas
42 mins ago


















draft saved

draft discarded




















































Thanks for contributing an answer to Mathematics Stack Exchange!


  • Please be sure to answer the question. Provide details and share your research!

But avoid



  • Asking for help, clarification, or responding to other answers.

  • Making statements based on opinion; back them up with references or personal experience.


Use MathJax to format equations. MathJax reference.


To learn more, see our tips on writing great answers.




draft saved


draft discarded














StackExchange.ready(
function () {
StackExchange.openid.initPostLogin('.new-post-login', 'https%3a%2f%2fmath.stackexchange.com%2fquestions%2f3201256%2fprove-the-alternating-sum-of-a-decreasing-sequence-converging-to-0-is-cauchy%23new-answer', 'question_page');
}
);

Post as a guest















Required, but never shown





















































Required, but never shown














Required, but never shown












Required, but never shown







Required, but never shown

































Required, but never shown














Required, but never shown












Required, but never shown







Required, but never shown







Popular posts from this blog

Щит и меч (фильм) Содержание Названия серий | Сюжет |...

is 'sed' thread safeWhat should someone know about using Python scripts in the shell?Nexenta bash script uses...

Meter-Bus Содержание Параметры шины | Стандартизация |...